LSAT and Law School Admissions Forum

Get expert LSAT preparation and law school admissions advice from PowerScore Test Preparation.

 Administrator
PowerScore Staff
  • PowerScore Staff
  • Posts: 8916
  • Joined: Feb 02, 2011
|
#41246
Complete Question Explanation
(The complete setup for this game can be found here: lsat/viewtopic.php?t=11881)

The correct answer choice is (A)

S—the one random in the game—can only be in space 2 in the scenario in Template #2:

PT66_J12_Game_#2_#7_diagram 1.png
This information is sufficient to eliminate answer choices (B), (C), and (D), leaving just answer choices (A) and (E) in contention.

In examining answer choices (A) and (E), consider that only R2, T, and the remainder of O/V have yet to be placed. T is the most problematic of those variables because of the TV not-block created by the fourth rule. Thus, you should expect that the fourth rule will play a role in differentiating the two remaining answers.

In answer choice (A), placing O in space 5 forces V into space 6. R2 must then be in space 4, leaving space 3 for T:
PT66_J12_Game_#2_#7_diagram 2.png
Because O could be in space 5, answer choice (A) is thus correct.

In answer choice (E), placing V in space 4 forces O to be in space 6. R2 must then be in space 5, leaving only space 3 for T. However, this forces T and V to be next to each other, a violation of the fourth rule. Thus, answer choice (E) cannot be true and is incorrect.

Get the most out of your LSAT Prep Plus subscription.

Analyze and track your performance with our Testing and Analytics Package.